Bạn chưa đăng nhập. Vui lòng đăng nhập để hỏi bài

Những câu hỏi liên quan
dia fic
Xem chi tiết
Lê Hoàng Hiếu
Xem chi tiết
Nguyễn Việt Lâm
1 tháng 11 2021 lúc 23:25

\(S=\dfrac{1}{a^3+b^3}+\dfrac{1}{a^2b}+\dfrac{1}{ab^2}\ge\dfrac{1}{a^3+b^3}+\dfrac{4}{a^2b+ab^2}\)

\(S\ge\left(\dfrac{1}{a^3+b^3}+\dfrac{1}{a^2b+ab^2}+\dfrac{1}{a^2b+ab^2}+\dfrac{1}{a^2b+ab^2}\right)+\dfrac{1}{ab\left(a+b\right)}\)

\(S\ge\dfrac{16}{a^3+b^3+3a^2b+3ab^2}+\dfrac{1}{\dfrac{\left(a+b\right)^2}{4}.\left(a+b\right)}=\dfrac{20}{\left(a+b\right)^3}\ge20\)

\(S_{min}=20\) khi \(a=b=\dfrac{1}{2}\)

PhamQuangLocAAA
Xem chi tiết
meme
26 tháng 8 2023 lúc 7:15

Để tìm giá trị nhỏ nhất của biểu thức B = ab + bc + ca + a^3 + b^3 + c^3 / 5(ab + bc + ca) + 1, ta có thể sử dụng phương pháp đạo hàm.

Đầu tiên, ta tính đạo hàm của biểu thức B theo a, b và c. Đạo hàm riêng của B theo a, b và c được tính như sau:

∂B/∂a = 3a^2 + b^3 + c^3 / 5(ab + bc + ca) + 1 - (a^3 + b^3 + c^3)(b + c) / (5(ab + bc + ca) + 1)^2 ∂B/∂b = a^3 + 3b^2 + c^3 / 5(ab + bc + ca) + 1 - (a^3 + b^3 + c^3)(a + c) / (5(ab + bc + ca) + 1)^2 ∂B/∂c = a^3 + b^3 + 3c^2 / 5(ab + bc + ca) + 1 - (a^3 + b^3 + c^3)(a + b) / (5(ab + bc + ca) + 1)^2

Tiếp theo, ta giải hệ phương trình ∂B/∂a = ∂B/∂b = ∂B/∂c = 0 để tìm các điểm cực trị của biểu thức B.

Sau khi tìm được các điểm cực trị, ta so sánh giá trị của B tại các điểm cực trị và tại các điểm biên của miền xác định để tìm giá trị nhỏ nhất của B.

Tuy nhiên, việc giải phương trình và tính toán các giá trị có thể làm cho quá trình này trở nên phức tạp và mất nhiều thời gian.

Do đó, để tìm giá trị nhỏ nhất của biểu thức B, ta có thể sử dụng phương pháp khác như phương pháp đặt tính chất của hàm để giải quyết bài toán này.

Cody_Uni5
Xem chi tiết
Lê Ng Hải Anh
22 tháng 7 2018 lúc 16:08

Ta có: \(A=a^3+b^3+ab\)

\(=\left(a+b\right)\left(a^2-ab+b^2\right)+ab\)

\(=a^2-ab+b^2+ab\)

\(=a^2+b^2\)

Mà a+b=1 =>b=1-a

\(\Rightarrow A=a^2+\left(1-a\right)^2\)

\(=a^2+1-2a+a^2\)

\(=2a^2-2a+1\)

\(=2\left(a^2-a+\frac{1}{2}\right)\)

\(=2\left(a^2-a+\frac{1}{4}\right)+\frac{1}{2}\)

\(=2\left(a-\frac{1}{2}\right)^2+\frac{1}{2}\)

Ta có : \(2\left(a-\frac{1}{2}\right)^2\ge0\forall a\)

\(\Rightarrow2\left(a-\frac{1}{2}\right)^2+\frac{1}{2}\ge\frac{1}{2}\forall a\)

Hay: \(A\ge\frac{1}{2}\forall a\)

Dấu = xảy ra khi : \(2\left(a-\frac{1}{2}\right)^2=0\Rightarrow a=\frac{1}{2}\)

\(\Rightarrow b=1-\frac{1}{2}=\frac{1}{2}\)

Vậy MinA =1/2 tại a=b=1/2

Nguyễn Hoàng
22 tháng 7 2018 lúc 16:08

Đây mà toán lớp 9 

Cody_Uni5
22 tháng 7 2018 lúc 16:09

mk nhầm ok?

Lil Shroud
Xem chi tiết
Nguyễn Việt Lâm
16 tháng 9 2021 lúc 9:52

\(P=\dfrac{a^3}{b^2+ab+bc+ca}+\dfrac{b^3}{c^2+ab+bc+ca}+\dfrac{c^3}{a^2+ab+bc+ca}=\dfrac{a^3}{\left(a+b\right)\left(b+c\right)}+\dfrac{b^3}{\left(a+c\right)\left(b+c\right)}+\dfrac{c^3}{\left(a+b\right)\left(a+c\right)}\)

Ta có:

\(\dfrac{a^3}{\left(a+b\right)\left(b+c\right)}+\dfrac{a+b}{8}+\dfrac{b+c}{8}\ge\dfrac{3a}{4}\)

\(\dfrac{b^3}{\left(a+c\right)\left(b+c\right)}+\dfrac{a+c}{8}+\dfrac{b+c}{8}\ge\dfrac{3b}{4}\)

\(\dfrac{c^3}{\left(a+b\right)\left(a+c\right)}+\dfrac{a+b}{8}+\dfrac{a+c}{8}\ge\dfrac{3c}{4}\)

Cộng vế:

\(P+\dfrac{a+b+c}{2}\ge\dfrac{3}{4}\left(a+b+c\right)\)

\(\Rightarrow P\ge\dfrac{1}{4}\left(a+b+c\right)\ge\dfrac{1}{4}.\sqrt{3\left(ab+bc+ca\right)}=\dfrac{\sqrt{3}}{4}\)

Trịnh Thị Bình
Xem chi tiết
Lê Song Phương
Xem chi tiết
Xyz OLM
3 tháng 2 2023 lúc 21:37

1) Áp dụng bđt Cauchy cho 3 số dương ta có

 \(\dfrac{1}{x}+\dfrac{1}{x}+\dfrac{1}{x}+x^3\ge4\sqrt[4]{\dfrac{1}{x}.\dfrac{1}{x}.\dfrac{1}{x}.x^3}=4\) (1)

\(\dfrac{3}{y^2}+y^2\ge2\sqrt{\dfrac{3}{y^2}.y^2}=2\sqrt{3}\) (2)

\(\dfrac{3}{z^3}+z=\dfrac{3}{z^3}+\dfrac{z}{3}+\dfrac{z}{3}+\dfrac{z}{3}\ge4\sqrt[4]{\dfrac{3}{z^3}.\dfrac{z}{3}.\dfrac{z}{3}.\dfrac{z}{3}}=4\sqrt{3}\) (3)

Cộng (1);(2);(3) theo vế ta được

\(\left(\dfrac{3}{x}+\dfrac{3}{y^2}+\dfrac{3}{z^3}\right)+\left(x^3+y^2+z\right)\ge4+2\sqrt{3}+4\sqrt{3}\)

\(\Leftrightarrow3\left(\dfrac{1}{x}+\dfrac{1}{y^2}+\dfrac{1}{z^3}\right)\ge3+4\sqrt{3}\)

\(\Leftrightarrow P\ge\dfrac{3+4\sqrt{3}}{3}\)

Dấu "=" xảy ra <=> \(\left\{{}\begin{matrix}\dfrac{1}{x}=x^3\\\dfrac{3}{y^2}=y^2\\\dfrac{3}{z^3}=\dfrac{z}{3}\end{matrix}\right.\Leftrightarrow\left\{{}\begin{matrix}x=1\\y=\sqrt[4]{3}\\z=\sqrt{3}\end{matrix}\right.\) (thỏa mãn giả thiết ban đầu)

 

Xyz OLM
3 tháng 2 2023 lúc 22:03

2) Ta có \(4\sqrt{ab}=2.\sqrt{a}.2\sqrt{b}\le a+4b\)

Dấu"=" khi a = 4b

nên \(\dfrac{8}{7a+4b+4\sqrt{ab}}\ge\dfrac{8}{7a+4b+a+4b}=\dfrac{1}{a+b}\)

Khi đó \(P\ge\dfrac{1}{a+b}-\dfrac{1}{\sqrt{a+b}}+\sqrt{a+b}\)

Đặt \(\sqrt{a+b}=t>0\) ta được

\(P\ge\dfrac{1}{t^2}-\dfrac{1}{t}+t=\left(\dfrac{1}{t^2}-\dfrac{2}{t}+1\right)+\dfrac{1}{t}+t-1\)

\(=\left(\dfrac{1}{t}-1\right)^2+\dfrac{1}{t}+t-1\)

Có \(\dfrac{1}{t}+t\ge2\sqrt{\dfrac{1}{t}.t}=2\) (BĐT Cauchy cho 2 số dương)

nên \(P=\left(\dfrac{1}{t}-1\right)^2+\dfrac{1}{t}+t-1\ge\left(\dfrac{1}{t}-1\right)^2+1\ge1\)

Dấu "=" xảy ra <=> \(\left\{{}\begin{matrix}\dfrac{1}{t}-1=0\\t=\dfrac{1}{t}\end{matrix}\right.\Leftrightarrow t=1\)(tm)

khi đó a + b = 1

mà a = 4b nên \(a=\dfrac{4}{5};b=\dfrac{1}{5}\)

Vậy MinP = 1 khi \(a=\dfrac{4}{5};b=\dfrac{1}{5}\)

 

qqqqqqq
Xem chi tiết
dam quang tuan anh
9 tháng 11 2017 lúc 19:59

24+t94()
Xét hàm () được: MinF(t)=F(23)=19
MinP=MinF(t)=19.dấu "=" xảy ra khi a=b=c=13

secret_aquarius _
Xem chi tiết